Difference between revisions of "2006 AMC 10A Problems/Problem 4"

m (added link to previous and next problem)
m (added category)
Line 14: Line 14:
  
 
*[[2006 AMC 10A Problems/Problem 5|Next Problem]]
 
*[[2006 AMC 10A Problems/Problem 5|Next Problem]]
 +
 +
[[Category:Introductory Number Theory Problems]]

Revision as of 16:06, 4 August 2006

Problem

A digital watch displays hours and minutes with AM and PM. What is the largest possible sum of the digits in the display?

$\mathrm{(A) \ } 17\qquad \mathrm{(B) \ } 19\qquad \mathrm{(C) \ } 21\qquad \mathrm{(D) \ } 22\qquad \mathrm{(E) \ } 23$

Solution

The time with largest sum is 9:59.

$9+5+9=23,(E)$

See Also